Los elementos de la matriz de representación de posición del propagador de una partícula en un anillo

Tengo una duda sobre la obtención de elementos matriciales del operador de evolución temporal. Tengo el siguiente hamiltoniano para una partícula en un anillo con campo magnético

H = 2 2 metro R 2 [ i θ + ϕ mi h ] 2

y desde [ H , PAGS ] = 0 , por PAGS = i θ , tomé estados propios de momento como estados propios del hamiltoniano.

Luego expresé el operador de evolución temporal tu ( t 2 , t 1 ) = mi i H ( t 2 t 1 ) en la base propia del hamiltoniano de la siguiente manera

i H ( t 2 t 1 ) | norte = i ( t 2 t 1 ) mi norte | norte

i H ( t 2 t 1 ) norte = | norte norte | = i ( t 2 t 1 ) norte = mi norte | norte norte |

Y con la propiedad de completitud norte = | norte norte | = 1 obtenemos:

tu ( t 2 , t 1 ) = mi i H ( t 2 t 1 ) = norte = mi i ( t 2 t 1 ) mi norte | norte norte |

Ahora, sé que puedo intercalar el operador entre estados propios para encontrar elementos de la matriz. Pero quiero encontrar el elemento de matriz en el θ base. Más específicamente quiero encontrar θ 2 | tu ( t 2 , t 1 ) | θ 1 . Los autoestados del hamiltoniano en el θ base, θ | norte = 1 2 π mi i norte θ

Respuestas (1)

No lo pensarías, por lo fácil que es plantear esta pregunta, pero es ridículamente no trivial. Da la casualidad de que es completamente imposible encontrar los elementos de la matriz de base de posición de este propagador.

Hasta ahora lo has hecho bien, y la identificación

tu ( t 2 , t 1 ) = mi i H ( t 2 t 1 ) = norte = mi i ( t 2 t 1 ) mi norte / | norte norte |
es correcto. Ingenuamente, el camino es claro, porque conoces las funciones propias,
θ | norte = 1 2 π mi i norte θ ,
así que, en principio, solo puedes hacer un sándwich tu ( t 2 , t 1 ) entre dos kets de posición y habrás terminado. esto te da
θ 2 | tu ( t 2 , t 1 ) | θ 1 = norte = mi i ( t 2 t 1 ) mi norte / θ 2 | norte norte | θ 1 = 1 2 π norte = mi i ( t 2 t 1 ) mi norte / mi i norte ( θ 1 θ 2 ) ,
y todo se ve bien, ¿verdad? Solo tienes que calcular una serie y estarás en camino. Además, puede simplificar un poco las cosas aquí: puede establecer θ = θ 1 θ 2 y τ = t 2 t 1 para simplificar la notación (es decir, el propagador solo depende de esas diferencias, lo cual es bueno), y también puede calcular las energías,
mi norte = 2 2 metro R 2 ( norte + mi h ϕ ) 2 ,
de modo que ( t 2 t 1 ) mi norte = 2 metro R 2 ( norte + mi h ϕ ) 2 τ = k norte 2 τ + λ norte ϕ τ + m ϕ 2 τ , y todo esto lo convierte en una serie muy simplificada:
θ 2 | tu ( t 2 , t 1 ) | θ 1 = mi i m ϕ 2 τ 2 π norte = mi i norte 2 k τ mi i norte ( θ λ ϕ τ ) ,
y esto es limpio y simple como puede ser. ¿Qué podría salir mal?


Ahora, da la casualidad de que esta serie es exactamente sumable y representa algo llamado Jacobi ϑ 3 función , y puede leer todo sobre ella en el DLMF . Más precisamente, en este caso, la relación se reduce a

( ) θ 2 | tu ( t 2 , t 1 ) | θ 1 = 1 4 π i k τ mi i m ϕ 2 τ mi i ( λ ϕ τ θ ) 2 4 k τ ϑ 3 ( π 2 k τ ( λ ϕ τ θ ) , mi i π 2 k τ ) ,
y parece que has terminado: has reducido tu serie a alguna función especial, y obviamente no puedes reducirla más, así que no hay nada más que hacer.

Desafortunadamente, sin embargo, aún no has terminado. Si observa más de cerca la función theta, normalmente se define como

ϑ 3 ( z , q ) = 1 + 2 norte = 0 q norte 2 porque ( 2 norte z ) ,
dónde q y z son parámetros complejos, y querremos establecer q = mi i π 2 / k τ . Es obvio a partir de la definición que la serie convergerá si | q | < 1 , y que divergirá si | q | > 1 . El resultado de la mecánica cuántica que acabamos de obtener está equilibrado en el filo de la navaja, en | q | = 1 : no hay garantías de convergencia, pero la serie es oscilatoria, por lo que tal vez podamos esperar alguna convergencia condicional provocada por las oscilaciones.

Por desgracia, tampoco hay alegría allí. La respuesta completa es que ϑ 3 tiene algo llamado límite natural , lo que significa que para cualquier punto q 0 en el círculo unitario (entonces | q 0 | = 1 ) y cualquier z C , el límite límite q q 0 ϑ 3 ( z , q ) diverge hasta el infinito. (Estoy usando la notación q q 0 dar a entender que q permanece dentro del disco durante el límite, solo para mantener las cosas fáciles). Si desea conocer los grandes detalles matemáticos feos, esto se debe a que la serie es algo llamado serie lacunaria , sobre la cual pregunté aquí en MathOverflow y aquí en Maths SE; en particular, aquí el teorema de la brecha de Fabry implica que la función theta tiene un anillo de fuego a su alrededor, que se ve así:

Código de Mathematica a través de Import[" https://raw.githubusercontent.com/halirutan/Mathematica-SE-Tools/master/SETools/SEImageExpressionDecode.m "][" http://i.stack.imgur.com/wM9dN.png "]

El problema aquí es que nuestro ϑ 3 está sentado justo encima del límite natural: es una función analítica para | q | < 1 , pero es probable que no haya forma de proporcionar una continuación analítica más allá de su círculo de convergencia (y eso es un fastidio), y probablemente no haya forma de darle algún sentido para | q | = 1 , que es exactamente donde estamos.


¿Entonces, dónde nos deja eso? Nuestro buen propagador ( ) parecía lo suficientemente limpio, pero una inspección más cercana muestra que en realidad es una tontería matemática. ¿Qué significa esto y cómo lo solucionamos?

Bueno, hay un par de formas, pero ambas esencialmente significan que solo puedes interpretar θ 2 | tu ( t 2 , t 1 ) | θ 2 + θ como algo llamado función generalizada , también conocida como distribución : un objeto, algo así como la función delta de Dirac, que no tiene sentido por sí solo, sino solo cuando se integra en productos con otras funciones.

Hay dos maneras fáciles de hacer esto:

  • Una es tomar el complejo- q cosa en serio. Esto se logra fácilmente dando el tiempo τ = τ 0 i d una componente imaginaria distinta de cero, que trae | q | hasta debajo 1 , y deja θ 2 | tu ( t 2 , t 1 ) | θ 1 tener sentido.

    Si hace esto, la clave a tener en cuenta es que normalmente las únicas cantidades físicamente significativas donde usará el propagador son elementos de matriz de la forma F | tu ( t 2 , t 1 ) | gramo para estados físicos | F y | gramo . Esos finalmente se reducen a integrales de la forma

    F | tu ( t 2 , t 1 ) | gramo = 0 2 π 0 | tu ( t 2 , t 1 ) | θ F ( θ ) d θ = 0 2 π mi i m ϕ 2 τ mi i ( λ ϕ τ θ ) 2 4 k τ 4 π i k τ ϑ 3 ( π 2 k τ ( λ ϕ τ θ ) , mi i π 2 k τ ) F ( θ ) d θ ,
    y aquí la receta dice que debes leer esas integrales como el límite de integrales con un distinto de cero d como eso d va a cero:
    F | tu ( t 2 , t 1 ) | gramo = límite d 0 0 2 π mi i m ϕ 2 τ mi i ( λ ϕ τ θ ) 2 4 k τ 4 π i k τ ϑ 3 ( π 2 k τ ( λ ϕ τ θ ) , mi i π 2 / k ( τ i d ) ) F ( θ ) d θ .
    Si haces eso, lo que pasa es que para cada d el integrando está bien definido, es analítico y se comporta perfectamente bien, y te da un resultado perfectamente normal para la integral. Más importante aún, esos resultados se acercarán a un límite a medida que d 0 mientras F ( θ ) no es absolutamente loco.

  • El otro enfoque es similar, pero está basado en series, es decir, requiere que tome nuestra serie original antes de la evaluación para ϑ 3 ,

    θ 2 | tu ( t 2 , t 1 ) | θ 1 = mi i m ϕ 2 τ 2 π norte = mi i norte 2 k τ mi i norte ( θ λ ϕ τ ) ,
    y verlo como una indicación de que necesita tomar el | norte | límite después de haber hecho cualquier producto interno relevante que necesite tomar. Para la integral anterior, por ejemplo, esto significa entender que
    0 2 π 0 | tu ( t 2 , t 1 ) | θ F ( θ ) d θ = mi i m ϕ 2 τ 2 π norte = mi i norte 2 k τ 0 2 π mi i norte ( θ λ ϕ τ ) F ( θ ) d θ ,
    con la serie después de la integral. Si F ( θ ) se comporta bien, entonces sus coeficientes de la serie de Fourier disminuirán con | norte | razonablemente rápido, y la serie (¡vaya!) convergerá nuevamente, como debe ser para un elemento de matriz físicamente relevante.

  • Alternativamente, puede tomar la expresión en serie que teníamos antes,

    θ 2 | tu ( t 1 + τ , t 1 ) | θ 2 + θ = mi i m ϕ 2 τ 2 π norte = mi i norte 2 k τ mi i norte ( θ λ ϕ τ ) ,
    y simplemente verlo como una identidad entre distribuciones.

    Este paradigma requiere que vea cada mapeo θ mi i norte 2 k τ mi i norte ( θ λ ϕ τ ) no como una función sino como una distribución, que luego sumas como una serie. Esto es algo bastante fuerte, ya que requiere que dejes de ver θ 2 | norte norte | θ 1 como algo que tiene un valor si le das un valor específico θ 1 , θ 2 y norte , pero si haces esto obtienes algunas buenas ventajas matemáticas.

    Más específicamente, cada una de las distribuciones θ D S mi i norte 2 k τ mi i norte ( θ λ ϕ τ ) es una distribución temperada (es decir, el tipo de distribución que funciona bien con las transformadas de Fourier y la mecánica cuántica; véanse estas notas , estas o estas para obtener más detalles), y la serie está delimitada arriba por el peine de Dirac , que también es una distribución temperada, y esto es ( ¿probablemente? ) suficiente para mostrar que la serie suma una distribución temperada.

    Por lo tanto, en este paradigma, obtienes que el propagador existe y es tan bueno como te gustaría que fuera (dentro de los límites de que es una distribución) y que, de hecho, es el límite de la serie en el lado derecho. , siempre y cuando lo tomes en el sentido distributivo.


Eso es más o menos, realmente, y creo que esto es tan claro como puedo hacerlo. Cerraré con las dos referencias donde aprendí estas cosas,

L. Schulman. Una integral de trayectoria para espín. física Rev. 176 no. 5, pág. 1558. (1968) .

y

LS Schulman. Técnicas y aplicaciones de la integración de caminos (Dover, Nueva York, 2005), pp. 190-196.

y finalmente con una cita de Schulman (realmente presente en ambas referencias), que describe la situación aquí con bastante precisión:

El grado de patología exhibido por esta función de Green es entretenido, especialmente en vista de la naturaleza elemental del ejemplo.

¿Desde entonces te has topado con alguna idea física de por qué las matemáticas resultan ser tan patológicas? ¿Es el ejemplo de alguna manera demasiado artificial? En casos bastante simples, siento que a veces las divergencias surgen de idealizaciones (¿como ignorar la reacción inversa en el campo magnético?), Pero aquí parece que asignar dinámicas al campo magnético convocaría las cincuenta sombras del infinito del QED :/ :(
No, no lo he hecho, lamento decirlo. He mostrado esto un par de veces en el contexto de una conferencia, con algunas personas inteligentes asistiendo, y no surgió una idea más profunda. Tal vez valga la pena tener una pregunta separada sobre por qué ocurre este comportamiento; podría incitar a los mecánicos Q y V Morettis del mundo a examinarlo más de cerca y potencialmente encontrar una buena explicación.